12
$\begingroup$

Let $\mathbb{R}$ be the real field. For any homogeneous polynomial $f(X_1,\cdots,X_n)$ in $\mathbb{R}[X_1,\cdots,X_n]$, we use $S_f(X_1,\cdots,X_n)$ to denote the following homogeneous symmetric polynomial: $$S_f(X_1,\cdots,X_n)=\sum_{\sigma=[i_1,\cdots,i_n]\in S_n}f(X_{i_1},\cdots,X_{i_n}).$$ Here the sum is computed over all permutations $\sigma=[i_1,\cdots,i_n](\sigma(1)=i_1,\cdots,\sigma(n)=i_n)$ of the set $\{1,\cdots,n\}$ and the set of all such permutations is denoted $S_n.$ We say $f(X_1,\cdots,X_n)$ is good if $$S_f(a_1,\cdots,a_n)\geq 0$$ for every $(a_1,\cdots,a_n)\in \mathbb{R}^n.$ For example, when $n=3$, we have $$f(X_1,X_2,X_3)=X_3^2-X_1X_2$$ is good because$$S_f(X_1,X_2,X_3)=(X_1-X_2)^2+(X_2-X_3)^2+(X_3-X_1)^2.$$ For any $n\geq 1$, define the homogeneous polynomial of degree $n^2$ in $\mathbb{R}[X_1,X_2,\cdots,X_{2n}]$ as follow:$$\varphi_n(X_1,X_2,\cdots,X_{2n})=\prod_{\substack{1\leq i\leq n\\n+1\leq j\leq 2n}}(X_i-X_j).$$ I conjecture that $\varphi_n$ is good for any $n\geq 1$, and for this conjecture I have got the following simple results:

$(1)$It is easy to proof that $S_{\varphi_n}(X_1,X_2,\cdots,X_{2n})=0$ when $n$ is odd;

$(2)$When $n$ is even,$$S_{\varphi_2}(X_1,X_2,X_3,X_4)=4[((X_1-X_2)(X_3-X_4))^2+((X_1-X_3)(X_2-X_4))^2+((X_1-X_4)(X_2-X_3))^2].$$ So the conjecture is right for $n=2$.

I can not proof it any more for $n\geq 4$, but I believe that the conjecture is right. Would you please give me some help?

$\endgroup$
4
  • 1
    $\begingroup$ Case (1) is reminiscent of en.wikipedia.org/wiki/Amitsur%E2%80%93Levitzki_theorem $\endgroup$
    – Suvrit
    Dec 20, 2015 at 15:56
  • 2
    $\begingroup$ @Suvrit Case (1) is completely obvious: if we interchange sets $\{x_1,\dots,x_n\},\{x_{n+1},\dots,x_{2n}\}$, the product changes its sign. $\endgroup$ Dec 20, 2015 at 16:34
  • 1
    $\begingroup$ @FedorPetrov thanks, it is indeed obvious. I had just remarked about that to make a connection with a well-known symmetric polynomial identity :-) $\endgroup$
    – Suvrit
    Dec 20, 2015 at 17:28
  • $\begingroup$ related question mathoverflow.net/questions/107526/… $\endgroup$ Mar 26, 2016 at 16:42

1 Answer 1

11
$\begingroup$

Alas, this is false at least for even $n\geqslant 6$, I do not know about $n=4$.

A similar question about symmetrization of $$f_{a,b}:=\prod_{1\leq i\leq a,a+1\leq j \leq a+b} (X_i-X_j)$$ may be asked (of course, it equals to 0 if $ab$ is odd). I asked this for $a=1$, $b=n-1$ here and it turned out to fail for odd $n\geq 7$.

Now for your question. Take even $n\geq 6$ and $n-1$ variables equal taking very-very large value $M$, and some $n+1$ variables $X_1,\dots,X_{n+1}$. Then if we divide the value $$f_{n,n}(M,M,\dots,M,X_{1},X_{2},\dots,X_{n+1})$$ by $M^{n(n-1)}$, it tends to $$f_{1,n}(X_1,\dots,X_{n+1}),$$ therefore if symmetrization of the latter is negative, symmetrization of the former is negative for large enough $M$.

It would be interesting to describe all pairs $(a,b)$ for which inequality holds.

$\endgroup$
1
  • $\begingroup$ Fedor Petrov: Thank you very much for your help! $\endgroup$
    – user173856
    Dec 22, 2015 at 1:53

Your Answer

By clicking “Post Your Answer”, you agree to our terms of service and acknowledge you have read our privacy policy.

Not the answer you're looking for? Browse other questions tagged or ask your own question.